Double Integral: ∫∫D x^2 + y^2 dA for D limited by: y=x^2, x=2, y=1

In summary, the student attempted to solve a homework equation involving the region D, but did not provide a clear or coherent explanation of their method.
  • #1
WrongMan
149
15

Homework Statement


∫∫D x2+y2dA
where D is the region limited by: y=x2, x=2, y=1 (dA = dxdy)

Homework Equations

The Attempt at a Solution


So basically i sketched the area, and i get the area under y=x2 0<x<1 and a square at 1<x<2 , 0<y<1
So i divded the integrals; for the square ∫0112 of x2+y2dxdy
for the y=x2 part is the part i need help/confirming x goes from 0 to 1(so integral os x2 with those bounds), and i used substitution for y=x2 and got dy=2xdx so integrated from 0 to 1 x42x dx

So i was thinking about this in another way, which gave me the same answer.
So i integrated x2 and kept the bounds so i got [(x3)/3]10 then integrated y2 and set its bounds to 0 to x2 and substituted so i got [(x3)/3]10 + x^5/3, and moved that inside the bounds.
Am i correct?
 
Last edited:
Physics news on Phys.org
  • #2
WrongMan said:

Homework Statement


∫∫D x2+y2dA
where D is the region limited by: y=x2, x=2, y=1 (dA = dxdy)

Homework Equations

The Attempt at a Solution


So basically i sketched the area, and i get the area under y=x2 0<x<1 and a square at 1<x<2 , 0<y<1
So i divded the integrals; for the square ∫0112 of x2+y2dxdy
for the y=x2 part is the part i need help/confirming x goes from 0 to 1(so integral os x2 with those bounds), and i used substitution for y=x2 and got dy=2xdx so integrated from 0 to 1 x42x dx
I have a different interpretation of the region D.

You can always plot these in utilities like Wolfram Alpha.

I take the upper boundary of D to be y = x2.

The right hand boundary is going to be the vertical line x = 2.

The lower boundary will be the horizontal line y = 1. This will also produce the left-hand limit of the region in terms of the x-coordinate, where y = 1 intersects y = x2.

The region D is roughly in the shape of a triangle, with the hypotenuse being a segment of the parabola y = x2 and the base being the line y = 1.


https://www4f.wolframalpha.com/Calculate/MSP/MSP641hbce22d448geh87000055267hbac324i9hi?MSPStoreType=image/gif&s=47 [Broken]

As far as the mechanics of your integration goes, I have no idea what you are trying to do. You don't have the correct region for D and your substitution is frankly a head-scratcher.

In any event, since you have the double integral of a sum, it would probably be easier to split this into the sum of the integrals of x2 and y2 dA and add the results:

##\int \int_D (x^2+y^2)\, dA = \int \int_D x^2\,dA + \int\int_D y^2\, dA##
 
Last edited by a moderator:
  • Like
Likes WrongMan
  • #3
The best way to perform this double integral is to do the dy integral first. The dx integral (done second) has two parts that will have different boundaries. So basically divide the double integral into two sections: 1) ## 0<x<1 ## and 2) ## 1<x<2 ##. The dx integral from 0 to 1 is bounded by ## y=x^2 ## for the portion 0<x<1 (as you correctly noticed), and by y=1 for ## 1<x<2 ##. For the first section , you need to carefully look at the dy limits. Do you see that the upper limit here (on the dy integration) needs to be ## x^2 ##? It does take a little effort to get some proficiency with these double integrals... editing...sorry, my mistake=I see I got the boundary region incorrect. I mistakenly thought you had y=0 as a lower boundary...but I see, yes, I didn't read your problem statement and assumed you had the region correct...
 
Last edited:
  • Like
Likes WrongMan
  • #4
ok so my first attempt was incorrect, i see that now...

The problem with my second attempt was that my teacher defined dA as dxdy so i can't integrate y first, so i have to take the lower limit of x as square root of y.
If i could integrate dy first i could've done what i did in my second attempt and set the upper limmit of y as x2

Am i correct?

so using the now correct boundries ( I am leaving the 1<x<2 part out)
01y½1 x2+y2 dxdy
so i get
011/3- (1/3*y3/2)+y2 dy

is this correct?
 
Last edited:
  • #5
WrongMan said:
ok so my first attempt was incorrect, i see that now...

The problem with my second attempt was that my teacher defined dA as dxdy so i can't integrate y first, so i have to take the lower limit of x as square root of y.
If i could integrate dy first i could've done what i did in my second attempt and set the upper limmit of y as x2

Am i correct?

so using the now correct boundries ( I am leaving the 1<x<2 part out)
01y½1 x2+y2
so i get
011/3- (1/3*y3/2)+y2 dy

is this correct?
Who says you can't integrate w.r.t. y first? dA = dxdy is the same as dA = dydx. In fact, you should get the same answer to the integral regardless of which variable you choose first.
 
  • Like
Likes WrongMan
  • #6
SteamKing said:
Who says you can't integrate w.r.t. y first? dA = dxdy is the same as dA = dydx. In fact, you should get the same answer to the integral regardless of which variable you choose first.
in this particular set of problems my teacher said to use dA = dxdy, so i can't use dydx
 
  • #7
WrongMan said:
in this particular set of problems my teacher said to use dA = dxdy, so i can't use dydx
I don't know why this is, but you should be able to use any order to integrate your function. At least, in the real world you can.
 
  • Like
Likes WrongMan
  • #8
SteamKing said:
I don't know why this is, but you should be able to use any order to integrate your function. At least, in the real world you can.
i'm guessing that its for us to practice integration limits.
All the other problems i can use whatever i want for dA
 
  • #9
WrongMan said:
ok so my first attempt was incorrect, i see that now...

The problem with my second attempt was that my teacher defined dA as dxdy so i can't integrate y first, so i have to take the lower limit of x as square root of y.
If i could integrate dy first i could've done what i did in my second attempt and set the upper limmit of y as x2

Am i correct?

so using the now correct boundries ( I am leaving the 1<x<2 part out)
01y½1 x2+y2 dxdy
so i get
011/3- (1/3*y3/2)+y2 dy

is this correct?
Not completely correct. Your upper dx limit needs to be 2. And your y limits are incorrect.
 
  • Like
Likes WrongMan
  • #10
Charles Link said:
Almost correct. Your upper dx limit needs to be 2.
i was doing that on a separate integral, since i get a square from 1<x<2 and 0<y<1, I am suming both up, the one on here, and the double integral with bounds 1-2 on dx and 0 to 1 on dy.
I can do that right?
 
  • #11
WrongMan said:
i was doing that on a separate integral, since i get a square from 1<x<2 and 0<y<1, I am suming both up, the one on here, and the double integral with bounds 1-2 on dx and 0 to 1 on dy.
I can do that right?
Please read SteamKing post #2 and also my edited post #3. Your boundary is a triangular shaped region. (See his graph). Also see my edited post #9.
 
  • Like
Likes WrongMan
  • #12
ohhh so i thought the lower limit was y=1 as shown in the pic. you are telling me its only the area marked (1) ?
 

Attachments

  • 1466191226322-210294188.jpg
    1466191226322-210294188.jpg
    45.5 KB · Views: 394
Last edited:
  • #13
WrongMan said:
ohhh so i thought the lower limit was y=1 as shown in the pic. you are telling me its only the area marked (1) ?
Yes...The other calculation would make for an interesting exercise also, but it's not what the problem asked for... The lower limit is y=1, not y=0. With the new area, try to write out the limits. You're on the right track=you simply selected the wrong region...
 
Last edited:
  • Like
Likes WrongMan
  • #14
Thanks for your help guys.
I also realized i forgot to integrate y2 with respect to x getting y2x

One last question, when no lower bound is mentioned am i to take y=0 as a lower bound?

and just to check if i have this right, another problem states that the area is limited by y>=x2 and y<2x
so the lower limit on the dx integral would be y/2 and the upper would be square root of y. correct?
and the dy is from 0 to 2 ?
 
  • #15
WrongMan said:
Thanks for your help guys.
I also realized i forgot to integrate y2 with respect to x getting y2x

One last question, when no lower bound is mentioned am i to take y=0 as a lower bound?

and just to check if i have this right, another problem states that the area is limited by y>=x2 and y<2x
so the lower limit on the dx integral would be y/2 and the upper would be square root of y. correct?
and the dy is from 0 to 2 ?
When you previously had ## y^2 ## there after the dx integral, I thought you might have already put ## y^2 x ## and evaluated it because your x limits were 0 and 1. For your problem where y<2x, draw the line y=2x and it is the region below it, and above the curve ## y=x^2 ##. It's a crescent shape between x=0 and x=2. The dy is from 0 to 4. (assuming you are doing dx first, yes I think you got te dx limits correct.)
 
  • Like
Likes WrongMan
  • #16
Charles Link said:
When you previously had ## y^2 ## there after the dx integral, I thought you might have already put ## y^2 x ## and evaluated it because your x limits were 0 and 1. For your problem where y<2x, draw the line y=2x and it is the region below it, and above the curve ## y=x^2 ##. It's a crescent shape between x=0 and x=2. The dy is from 0 to 4. (assuming you are doing dx first, yes I think you got te dx limits correct.)

oh yeah i meant dy 0 to 4 got mixed up there.

yeah i sketched it, and noticed that as dx "grew" it was limited by y=x^2 , and when dx" went to 0" it was limited by y=2x, so that's where i got lower bound y/2 and upper bound sqrt(y)

and yes for this set of problems i got to do dx first, i don't get to chose.
I think I am getting the hang of it. Thanks!
 
  • Like
Likes Charles Link
  • #17
WrongMan said:
Thanks for your help guys.
I also realized i forgot to integrate y2 with respect to x getting y2x

One last question, when no lower bound is mentioned am i to take y=0 as a lower bound?

and just to check if i have this right, another problem states that the area is limited by y>=x2 and y<2x
so the lower limit on the dx integral would be y/2 and the upper would be square root of y. correct?
and the dy is from 0 to 2 ?
Whew! That's a confusing way to describe the region of interest.

Let's just say that you draw the line y = 2x and the parabola y = x2.

You want the area of the region which is between the two curves, so along a line of constant x-value, y ≥ x2 and y < 2x
 
  • Like
Likes WrongMan
  • #18
WrongMan said:
Thanks for your help guys.
I also realized i forgot to integrate y2 with respect to x getting y2x

One last question, when no lower bound is mentioned am i to take y=0 as a lower bound?

and just to check if i have this right, another problem states that the area is limited by y>=x2 and y<2x
so the lower limit on the dx integral would be y/2 and the upper would be square root of y. correct?
and the dy is from 0 to 2 ?

If no lower bound is mentioned there is no justification for taking y = 0 as a lower bound; basically -∞ is your lower bound, and that would make your integration come out infinite because the integration region is unbounded.

I think that the only reasonable interpretation of this problem is that the integration region is like a triangle with a curved hypotenuse, like the following shaded region.

upload_2016-6-17_13-45-48.png
 
  • Like
Likes WrongMan
  • #19
SteamKing said:
Whew! That's a confusing way to describe the region of interest.

Let's just say that you draw the line y = 2x and the parabola y = x2.

You want the area of the region which is between the two curves, so along a line of constant x-value, y ≥ x2 and y < 2x

thats what i said... :D
 
  • #20
What does
##y=2x##​
have to do with anything in this problem?
 
  • #21
SammyS said:
What does
##y=2x##​
have to do with anything in this problem?

Nothing, sorry for the confusion.
Just read this post:
WrongMan said:
Thanks for your help guys.
I also realized i forgot to integrate y2 with respect to x getting y2x

One last question, when no lower bound is mentioned am i to take y=0 as a lower bound?

and just to check if i have this right, another problem states that the area is limited by y>=x2 and y<2x
so the lower limit on the dx integral would be y/2 and the upper would be square root of y. correct?
and the dy is from 0 to 2 4 ?
 
Last edited:
  • #22
WrongMan said:
...

One last question, when no lower bound is mentioned am i to take y=0 as a lower bound?
That would entirely depend upon the specifics of the problem.
 

1. What is a double integral?

A double integral is a type of integral that calculates the volume under a surface in a two-dimensional space. It involves integrating a function over a region in the x-y plane.

2. What does the notation ∫∫D represent?

The notation ∫∫D represents a double integral, where the inner integral is integrated with respect to y and the outer integral is integrated with respect to x.

3. How do you find the limits of integration for a double integral?

The limits of integration for a double integral are determined by the boundaries of the region D in the x-y plane. In this case, the region is limited by the parabola y = x^2, the line x = 2, and the line y = 1.

4. What is the function being integrated in this double integral?

The function being integrated is x^2 + y^2, which represents the volume under a parabolic surface in the given region D.

5. How do you evaluate a double integral?

To evaluate a double integral, you can use the method of iterated integrals, which involves integrating the inner integral first and then the outer integral. In this case, the limits of integration will be substituted into the function, and the integral can be solved using basic integral rules.

Similar threads

  • Calculus and Beyond Homework Help
Replies
3
Views
116
  • Calculus and Beyond Homework Help
Replies
7
Views
641
  • Calculus and Beyond Homework Help
Replies
5
Views
741
  • Calculus and Beyond Homework Help
Replies
1
Views
449
  • Calculus and Beyond Homework Help
Replies
6
Views
505
  • Calculus and Beyond Homework Help
Replies
4
Views
809
  • Calculus and Beyond Homework Help
Replies
13
Views
1K
  • Calculus and Beyond Homework Help
Replies
21
Views
2K
  • Calculus and Beyond Homework Help
Replies
24
Views
2K
  • Calculus and Beyond Homework Help
Replies
2
Views
459
Back
Top